Find the value of cos J rounded to the nearest hundredth, if necessary.

Find The Value Of Cos J Rounded To The Nearest Hundredth, If Necessary.

Answers

Answer 1

Answer:

cosJ ≈ 0.38

Step-by-step explanation:

This is a 5- 12- 13 right triangle

with HI = 12, HJ = 13, IJ = 5

cosJ = [tex]\frac{adjacent}{hypotenuse}[/tex] = [tex]\frac{IJ}{HJ}[/tex] = [tex]\frac{5}{13}[/tex] ≈ 0.38 ( to nearest hundredth )

Answer 2

The required value of Cos(J) for the given triangle is 0.38.

What is Pythagoras theorem?

Pythagoras theorem states that in a right angled triangle, the sum of the squares of the perpendicular and base is equal to the square of the hypotenuse. It can be written as h² = p² + b².

In the given ΔHIJ the sides are given as,

JH = 13  and HI = 12.

Now, apply Pythagoras theorem to obtain,

IJ² + HI² = JH²

⇒ IJ² = 13² - 12²

⇒ IJ = 5

Use trigonometric ratio to obtain,

Cos(J) = IJ/HI = 5/13 = 0.38

Hence, the value of Cos(J) is obtained as 0.38.

To know more about Pythagoras theorem click on,

https://brainly.com/question/343682

#SPJ2


Related Questions

could someone help im really bad at polynomials

Answers

Answer:2x^2 + 7x - 30

Step-by-step explanation:

1. x(2x-3)(x+3)^2

2. x(2x-3)(x+10)

3. 2x^2+10x-3x-30

4. 2x^2+7x-30

Two particles have positions at time t given by s1=4t-t^2 and s2=5t^2-t^3. Find the velocities

Answers

Step-by-step explanation:

Given that,

The positions of the first particle,[tex]s_1=4t-t^2[/tex]

Velocity,

[tex]v_1=\dfrac{ds_1}{dt}\\\\v_1=\dfrac{d(4t-t^2)}{dt}\\\\v_1=4-2t[/tex]

Position,

[tex]s_2=5t^2-t^3[/tex]

[tex]v_2=\dfrac{ds_2}{dt}\\\\v_2=\dfrac{d(5t^2-t^3)}{dt}\\\\v_2=10t-3t^2[/tex]

Hence, this is the required solution.

Click on the photo! Needing help ASAP please!♥️

Answers

A
This is like a base form of a prime polynomial
X^2+x+1
Answer: Choice A)  x^4+20x^2-100

=============================================================

Explanation:

Choice B has the GCF x we can factor out like so

10x^4-5x^3+70x^2+3x = x(10x^3-5x^2+70x+3)

Showing that choice B is not prime. If a polynomial can be factored, then we consider it not prime. It's analogous to saying a number like 15 isn't prime because 15 = 3*5, ie 15 can be factored into something that doesn't involve 1 as a factor.

In contrast, we consider 7 prime because even though 7 = 1*7, there aren't any other ways to write this integer as a factorization if we don't involve 1.

-----------------------------------

Choice C is a similar story. This time we can factor out 3

3x^2 + 18y = 3(x^2 + 6y)

So we can rule this out as well.

-----------------------------------

Choice D is a bit tricky, but we can use the difference of cubes factoring rule

a^3 - b^3 = (a-b)(a^2+ab+b^2)

where in this case a = x and b = 3y^2

Note how b^3 = (3y^2)^3 = 3^3*(y^2)^3 = 27y^(2*3) = 27y^6

All of this means choice D can be factored and it's not prime either.

------------------------------------

We've ruled out choices B through D. The answer must be choice A.

If you let w = x^2, then w^2 = x^4

The polynomial w^2+20w-100 is prime because setting it equal to zero and solving for w leads to irrational solutions. I'm assuming your teacher wants you to factor over the rational numbers.

Because w^2+20w-100 can't be factored over the rational numbers, neither can x^4+20x^2-100. This confirms that choice A is prime.

Can x3 – 3x + 1 be the quotient on division of x5+ 2x3 + x – 1 by a polynomial in x of degree 3? Justify​

Answers

Answer:

No

Step-by-step explanation:

x⁵ + 2x³ + x - 1  -> degree of  the polynomial is 5

So, when x⁵ is divided by x³, the quotient should be x⁵⁻³ = x².

So, x³ - 3x + 1 cannot be a quotient

A certain polygon has its vertices at the following points: (1, 1), (1, 8), (8, 1), and (8, 8)

Answers

Answer:

Can u explain more pls?

Step-by-step explanation:

Answer:

Square

Step-by-step explanation:

If 260 people donated $10.00 to an organization how much money would said organization have received in total?

Answers

Answer:

$ 2,600

Step-by-step explanation:

It's given that 260 People donated $10 is to an organisation . We need to find the total money received by the organisation .

Here we just need to the number of people with the amount each person gave , that is ;

[tex]\implies \rm Total \ amount = \$ 10 \times 260 \\\\\rm\implies Total \ amount = \$ 2,600 [/tex]

Hence the total money received by the organisation is $ 2600 .

Question 5 can I have as much as you can do please

Answers

Answer:

x = 12

Step-by-step explanation:

The sum of angles in a triangle is 180°

Thus,

(x + 20) + (10x) + (x + 16) = 180°

Collect like terms

x + 10x + x + 20 + 16 = 180°

12x + 36 = 180°

12x = 180 - 36

12x = 144

Divide both sides by 12

x = 144 ÷ 12

x = 12

Substitute the value of x into the question

Therefore,

x + 20 = 12 + 20

= 32°

10x = 10(12)

= 120°

x + 16 = 12 + 16

= 28°

Which of the following composition of transformations would create an
image that is not congruent to its original image?
A rotation of 45° followed by a reflection across the x-axis
A translation 2 units to the left followed by dilation by 1/2
A reflection across the y-axis followed by a rotation of 60°
A rotation of 135º followed by a translation of 4 units to the right

Answers

Since the shape won’t change. It won’t deform, nothing will happen to the interior angle.

So the interior angle would still be 135

The Marked price of an article was fixed to Rs 1380 by increasing 15% on its actual price. Find the actual price.​

Answers

Answer:

The actual price of the article was Rs. 1200.

Step-by-step explanation:

mp (marked price)

ap (actual price)

[tex]mp = 1380[/tex]

[tex]mp = ap + 15\%(ap)[/tex]

[tex]1 380 = ap + \frac{15}{100} ap[/tex]

[tex]1380 = \frac{100}{100} ap + \frac{15}{100} ap[/tex]

[tex]1380 = \frac{115}{100} ap[/tex]

[tex]1380 \div \frac{115}{100} = ap[/tex]

[tex]1380 \times \frac{100}{115} = ap[/tex]

[tex] \frac{138000}{115} = ap[/tex]

[tex]1200 = ap[/tex]

The actual price of the article is given by the equation A = $ 1,200

What is an Equation?

Equations are mathematical statements with two algebraic expressions flanking the equals (=) sign on either side.

It demonstrates the equality of the relationship between the expressions printed on the left and right sides.

Coefficients, variables, operators, constants, terms, expressions, and the equal to sign are some of the components of an equation. The "=" sign and terms on both sides must always be present when writing an equation.

Given data ,

Let the actual price of the article be A

Now , the equation will be

The marked price of the article be = $ 1380

The percentage of increase from the actual price = 15 %

So , the equation is

The actual price + percentage of increase from the actual price = 1380

Substituting the values in the equation , we get

A + ( 15/100 )A = 1380   be equation (1)

( 115/100 ) A = 1380

Multiply by 100 on both sides of the equation , we get

115A = 138000

Divide by 115 on both sides of the equation , we get

A = $ 1200

Therefore , the value of A is $ 1200

Hence , the actual price of the article is $ 1200

To learn more about equations click :

https://brainly.com/question/19297665

#SPJ2

Solve. Algebra 1
14=4-6x-2

Answers

Answer:

Step-by-step explanation:

resolver. Álgebra 1

14=4-6x-2

4-6x-2 : 2

Rpta: 2

On a coordinate plane, rhombus W X Y Z is shown. Point W is at (7, 2), point X is at (5, negative 1), point Y is at (3, 2), and point Z is at (5, 5).
What is the perimeter of rhombus WXYZ?

StartRoot 13 EndRoot units
12 units
StartRoot 13 EndRoot units
20 units

Answers

Answer:

[tex]P = 4\sqrt{13}[/tex]

Step-by-step explanation:

Given

[tex]W = (7, 2)[/tex]

[tex]X = (5, -1)[/tex]

[tex]Y = (3, 2)[/tex]

[tex]Z =(5, 5)[/tex]

Required

The perimeter

To do this, we first calculate the side lengths using distance formula

[tex]d = \sqrt{(x_2 - x_1)^2 + (y_2 - y_1)^2[/tex]

So, we have:

[tex]WX = \sqrt{(5- 7)^2 + (-1 - 2)^2[/tex]

[tex]WX = \sqrt{13}[/tex]

[tex]XY = \sqrt{(3-5)^2 + (2--1)^2}[/tex]

[tex]XY = \sqrt{13}[/tex]

[tex]YZ = \sqrt{(5-3)^2 + (5-2)^2}[/tex]

[tex]YZ = \sqrt{13}[/tex]

[tex]ZW = \sqrt{(7 - 5)^2 + (2 - 5)^2}[/tex]

[tex]ZW = \sqrt{13}[/tex]

The perimeter is:

[tex]P = WX + XY + YZ + ZW[/tex]

[tex]P = \sqrt{13}+\sqrt{13}+\sqrt{13}+\sqrt{13}[/tex]

[tex]P = 4\sqrt{13}[/tex]

Answer:

C on edge 2021

Step-by-step explanation:

I took the cumulative exam

From the top of a building, 40 feet above the ground, a construction worker locates a rock at a 12° angle of depression. How far is the rock from the building? ​

Answers

Answer:

188 m

Step-by-step explanation:

[tex] \tan(12) = \frac{40}{x} \\ x = \frac{40}{ \tan(12) } \\ x = 188[/tex]

Answer:

I think that the other answer is WRONG

tan(12) = [tex]\frac{x}{40}[/tex]

x = tan(12)*40 = 8.5

Step-by-step explanation:

[tex]\frac{sin\left(78\right)}{40}=\frac{sin\left(12\right)}{x}[/tex] = 8.5

If a plane can travel 510 miles per hour with the wind and 410 miles per hour against the​ wind, find the speed of the wind and the speed of the plane in still air.​

Answers

Answer:

Below.

Step-by-step explanation:

Ok so, Since its 510 - x = 410. Speeds equals the wind.

SO?

100=2x

x=50

410+50 = 460!

A nunber when divided by 12 gives remainder 7 if the same number is divided by 6 the remainder must be ?

Answers

Answer:

1

Step-by-step explanation:

Let the number be n.

We are given:

n/12=q1+(7/12) where q1 is the quotient

n/6=q2+(?/6) where q2 is the quotient and ? is the remainder value we are trying to find.

? must be a integer between 0 and 5, inclusive. A remainder cannot be bigger than or equal to the divisor.

Let's rewrite the first equations

Multiply equation 1 on both sides by 2:

n/6=2q1+7/6

Remainder cannot be 7.

Rewrite again.

6 goes into 7 1 time with remainder 1.

n/6=2q1+(1+1/6)

n/6=(2q1+1)+1/6

So q2=2q1+1 and the remainder is 1 when dividing n by 6.

For fun. What is a number n with such conditions on it?

So what number has remainder 7 when divided by 12 and a remainder 1 when divided by 6.

n=12q1+7

n=6q2+1

If q1=1, we find a number right away that works.

19/12=1+7/12

19/6=3+1/6

Find the length of AC. Round to the nearest hundredth if necessary.

Answers

i belive the answer is c

The driving distance between Manchester and London is 195 miles. Farris wants to travel from Manchester to London on coach. The coach will leave Manchester at 3:30pm. Farris assumes the coach will travel at an average speed of 50mph. work out Farris's arrival time in London?

Answers

Answer:

7:24 pm

Step-by-step explanation:

speed = distance/time

time * speed = distance

time = distance/speed

time = (195 miles)/(50 miles/hour)

time = 3.9 hours

The trip will take 3.9 hours.

3 hours + 0.9 hours = 3 hours + 0.9 hours * 60 minutes/hour =

= 3 hours + 54 minutes

The trip will take 3 hours and 54 minutes.

3:30 pm + 3:54 = 6:84 pm = 7:24 pm

Answer: 7:24 pm

A solid wooden block in the shape of a rectangular prism has a length, width and height

Answers

Step-by-step explanation:

Volume of the block

[tex] = lbh[/tex]

[tex] = \frac{3}{8} \times \frac{1}{8} \times \frac{5}{8} [/tex]

[tex] = \frac{3 \times 5}{ {8}^{3} } [/tex]

[tex] = \frac{15}{512} [/tex]

Complete the function for this graph.

Answers

Answer:

[tex]y=|x-(-2)|+2[/tex]

Step-by-step explanation:

Hope this is helpful.

The required absolute function is given as y = |x - 2| + 2.

Given that,
From the graph, y = |x - h| + k, values of h and k is to be determined,

What are functions?

Functions are the relationship between sets of values. e g y=f(x), for every value of x there is its exists in a set of y. x is the independent variable while Y is the dependent variable.

Here,
In the graph and function, h represents the shift on the x-axis, which 2 units left so the value of h is 2, while k is given the shift of the function over the y-axis which 2 units up means that k = 2

Thus, The required absolute function is given as y = |x - 2| + 2.

learn more about function here:

brainly.com/question/21145944

#SPJ2

Joshua is 1.45 meters tall. At 2 p.m., he measures the length of a tree's shadow to be 31.65 meters. He stands 26.2 meters away from the tree, so that the tip of his shadow meets the tip of the tree's shadow. Find the height of the tree to the nearest hundredth of a meter .

Answers

Answer:

height of the tree ≈ 8.42 m

Step-by-step explanation:

The diagram given represents that of two similar triangles. Therefore, the corresponding lengths of the similar triangles are proportional to each other.

height of tree = h

Therefore:

1.45/h = (31.65 - 26.2)/31.65

1.45/h = 5.45/31.65

Cross multiply

h*5.45 = 1.45*31.65

h*5.45 = 45.8925

h = 45.8925/5.45

h ≈ 8.42 m (nearest hundredth)

What is the range of this set of heights in centimeters? {140, 166, 132, 165, 152, 168, 181, 158, 173, 171, 180, 182, 163, 177, 180, 142, 147, 149, 178} 38 41 46 50

Answers

Answer:

50

Step-by-step explanation:

Given:

140, 166, 132, 165, 152, 168, 181, 158, 173, 171, 180, 182, 163, 177, 180, 142, 147, 149, 178

Arranging in ascending order (from the lowest to the highest)

= 132, 140, 142, 147, 149, 152, 158, 163, 165, 166, 168, 171, 173, 177, 178, 180, 180 181, 182

Range = highest number - lowest number

= 182 - 132

= 50

Answer:

50

Step-by-step explanation:

              None

                     

             

             

 

         

Find the distance between each pair of points. Round to the nearest tenth if necessary.
(4,2) and (-6, -6)

Answers

Answer:

Radical (20)

Step-by-step explanation:

Radical ( (4-6)² + (2-6)²)) =radical ( 4+16) = radical (20)

Monica purchased adult and youth tickets for the fair. She bought x adult tickets for $28 each and y youth tickets for $15. Write an expression that can be
used to show the total cost, C, of all tickets.
O C = (28 + 15)x
OC = (28 + 15)(x + y)
O C = 28x + 15y
O C = 28y + 15x

Answers

Answer: =28x + 15y

Step-by-step explanation:

Write the trigonometric expression in terms of sine and cosine, and then simplify.

tan θ/(sec θ − cos θ)

Answers

Answer:

[tex]\displaystyle \frac{\tan\theta}{\sec\theta - \cos\theta} = \frac{1}{\sin\theta} = \csc\theta[/tex]

Step-by-step explanation:

We have the expression:

[tex]\displaystyle \frac{\tan\theta}{\sec\theta - \cos\theta}[/tex]

And we want to write the expression in terms of sine and cosine and simplify.

Thus, let tanθ = sinθ / cosθ and secθ = 1 / cosθ. Substitute:

[tex]=\displaystyle \frac{\dfrac{\sin\theta}{\cos\theta}}{\dfrac{1}{\cos\theta}-\cos\theta}[/tex]

Multiply both layers by cosθ:

[tex]=\displaystyle \frac{\left(\dfrac{\sin\theta}{\cos\theta}\right)\cdot \cos\theta}{\left(\dfrac{1}{\cos\theta}-\cos\theta\right)\cdot \cos\theta}[/tex]

Distribute:

[tex]\displaystyle =\frac{\sin\theta}{1-\cos^2\theta}[/tex]

Recall from the Pythagorean Theorem that sin²θ + cos²θ = 1. Hence, 1 - cos²θ = sin²θ. Substitute and simplify:

[tex]\displaystyle =\frac{\sin\theta}{\sin^2\theta} \\ \\ =\frac{1}{\sin\theta}[/tex]

We can convert this to cosecant if we wish.

Hi can i please get help on this question
Find the volume of this sphere.
Round to the nearest tenth.
16 ft
Formulas for Spheres

[? ] ft
Enter

Answers

Answer:

2144

formula us v = 3/4 × pi × half of the diameter (radius)

To solve the equation t? - t = 12 by factoring, you would use
t(t - 1) = 12
Ott - 1) - 12 = 0
(t - 4)(t + 3) = 0

Answers

the answer is : t= 4 , -3

Answer:

it is what it is

Step-by-step explanation:

Which relation is a function?

Answers

Answer:

the function is the solution is done using the operation

a function is where there is only one output for every input. so when you plug in an x value, you should be getting only one y value.

it’s not the first one because you can see there are two points at (-1,0)
it’s not the second one because you can see there are two points at (0,0)
it’s not the fourth one because again there are two points at (-2,0) and (2,0)

it’s the third one because there is only one y value for every x value.

Solve for x. Round to the nearest tenth of a degree, if necessary.

Answers

Answer:

x ≈ 60.6°

Step-by-step explanation:

Using the tangent ratio in the right triangle

tan x = [tex]\frac{opposite}{adjacent}[/tex] = [tex]\frac{IJ}{HI}[/tex] = [tex]\frac{3.9}{2.2}[/tex] , then

x = [tex]tan^{-1}[/tex] ([tex]\frac{3.9}{2.2}[/tex] ) ≈ 60.6° ( to the nearest tenth )

To make some draperies, an interior decorator needs 12 1/4 yards of material for the den and 8 1/2 yards for the living room. If the material comes only in 21-yard bolts, how much will be left over after completing both sets of draperies?

Answers

Answer:

1/4 of a yard of material

Step-by-step explanation:

First, we must convert both 12 1/4 yards and 8 1/2 yards to quarters. Since 1/2 of a yard is equal to 2/4 of a yard, 8 1/2 yards will be equal to 8 2/4 yards. 8 yards + 12 yards = 20 yards, and 2/4 of a yard + 1/4 of a yard = 3/4 of a yard, so you get 20 3/4 of a yard to finish the draperies. To find the leftover material, do 21 yards - 20 3/4 yards = 1/4 of a yard of material. Hope this helps!

Situation:
Find the age of
A student in Greece discovers a pottery
bowl that contains 28% of its original
amount of C-14.
Ent
N= Noekt
No
= inital amount of C-14 (at time
t = 0)
N = amount of C-14 at time t
k = 0.0001
t = time, in years

Answers

Answer:

Step-by-step explanation:

I'm assuming you need the age of the bowl. Start with the fact that you have remaining 28% of the original amount before any of it decayed. You always start with 100% of something unless you're told differently. That means that the equation looks like this:

[tex]28=100e^{-.0001t}[/tex] and begin by dividing both sides by 100 to get

[tex].28=e^{-.0001t}[/tex] . To solve for t we have to be able to bring it down from its current position of exponential. To do this we would either take the log or the natural log since the rules for both are the same. However, the natural log is the inverse of e, so they undo each other. We take the natural log of both sides which allows us to pull down the -.0001t. At the same time remember that the natural log and e are inverses of each other so they are both eliminated when we do this.

ln(.28) = -.0001t Now it's easy to solve for t.

[tex]\frac{ln(.28)}{-.0001}=t[/tex] and

[tex]\frac{-1.272965676}{-.0001}=t[/tex] so

t = 12729.65676 years or rounded, 12730 years.

One number is 96 more than another. Their ratio is 5:17. Find the numbers

Answers

Answer:

40 and 136

Step-by-step explanation:

The ratio of the 2 numbers = 5 : 17 = 5x : 17x ( x is a multiplier )

One number is 96 more than the other then

17x = 5x + 96 ( subtract 5x from both sides )

12x = 96 ( divide both sides by 12 )

x = 8 , then

5x = 5 × 8 = 40

17x = 17 × 8 = 136

The 2 numbers are 40 and 136

Other Questions
Holders of common stock in a corporation brought an action based on state law fraud claims arising out of a tender offer for their stock. The action against the corporation and its directors was filed in state court in State A. In entering a judgment against the corporation and its directors, the state court ruled that the tender offer contained false statements of material facts. Subsequently, holders of preferred stock in the corporation filed an action in federal district court in State B against the corporation and its directors, asserting a claim regarding the tender offer based on federal law. The preferred shareholders moved for a partial summary judgment on the issue of whether the tender offer contained false statements of material facts. Both State A and State B require mutuality of parties in order for issue preclusion to apply. Should the federal court grant the partial summary judgment motion Two linear equations are shown in the graph. #Brainliest award What are the coordinates of the point where the two lines intersect? A. (2, 3) B. (3, 3) C. (3, 0) D. (3, 3) Use a linear approximation (or differentials) to estimate the given number. (Round your answer to five decimal places.) 3 217 Which of the following is a base? (3 points)AgOCa(OH)2HFNaCl In Animal Farm, what does Squealer say will happen if the animals continue to have Sunday morning debates?Napoleon will make the animals work harder.Mr. Jones will come back to rule the farm.O The animals will have to build a windmill.O Snowball will return and make bad decisions. I need help with these questions do linear relationships have a constant rate of change Help me please guys Someone please help thanks Can you think of reasons why the charge on each ball decreases over time and where the charges might go Use a spelling word to complete the sentence.Clothing that is proper for an occasion may be called attire. A man received 7 per cent interest on a loan of $200 for 1 year. How much interest did he receive?This is making a tad confused, even though i'm sure its easy to solve. Find the value of "x" Wrong answer will be reported and explain please Synovec Co. is growing quickly. Dividends are expected to grow at a rate of 22 percent for the next three years, with the growth rate falling off to a constant 5 percent thereafter. If the required return is 12 percent, and the company just paid a dividend of $2.35, what is the current share price BRAINLIEST HELP ME!!!Which graph shows the solution to this system of linear Inequalities?ys2x-3A. GraphB. Graph BC. Graph AD. Graph D D = ______R = ______ Which non-mineral nutrient is essential for photosynthesis? ANSWER ASAP!!!!!!!!A.hydrogen B.potassium C.nitrogen D. carbon dioxide Which is true about this triangle? which property has not been observed for membrane proteins being degraded for energy during biological pathways Innovation is the introduction of